The closed form for $sum_{ngeq1}frac1{n(n+1)(n+2)cdots(n+m)}$ [duplicate]












2












$begingroup$



This question already has an answer here:




  • Sum of infinite series $sum_{i=1}^{infty} frac 1 {i(i+1)(i+2)…(i+n)}$

    4 answers




Just for fun, I defined the following series. For $minBbb N$,
$$g_m=sum_{ngeq1}prod_{i=0}^{m}frac1{n+i}$$
I am seeking a closed form for $g_m$. I found $g_1, g_2$, and a general expression involving a lot of integrals.



$$
begin{align}
g_1=&sum_{ngeq1}frac1{n(n+1)}\
=&sum_{ngeq1}frac1nint_0^1 t_0^nmathrm{d}t_0\
=&sum_{ngeq1}int_0^1int_0^{t_0} t_1^{n-1}mathrm{d}t_1mathrm{d}t_0 \
=&int_0^1int_0^{t_0}sum_{ngeq0}t_1^{n}mathrm{d}t_1mathrm{d}t_0 \
=&int_0^1int_0^{t_0}frac{mathrm{d}t_1}{1-t_1}mathrm{d}t_0 \
=&1 \
end{align}
$$

Through the same process, it can be shown that $g_2=frac14$. In fact, through the same process, it can be shown that
$$g_m=int_0^1int_0^{t_0}int_0^{t_1}cdotsint_0^{t_{m-2}}frac{mathrm{d}t_{m-1}}{1-t_{m-1}}mathrm{d}t_{m-2}cdotsmathrm{d}t_2mathrm{d}t_1mathrm{d}t_0$$
Which I don't even know how to approach. Any ideas? Thanks.



This isn't a duplicate question, because I would like to know how to evaluate those nested integrals, if possible.










share|cite|improve this question











$endgroup$



marked as duplicate by Math Lover, Lord Shark the Unknown, user10354138, Masacroso, Brahadeesh Nov 27 '18 at 5:51


This question has been asked before and already has an answer. If those answers do not fully address your question, please ask a new question.























    2












    $begingroup$



    This question already has an answer here:




    • Sum of infinite series $sum_{i=1}^{infty} frac 1 {i(i+1)(i+2)…(i+n)}$

      4 answers




    Just for fun, I defined the following series. For $minBbb N$,
    $$g_m=sum_{ngeq1}prod_{i=0}^{m}frac1{n+i}$$
    I am seeking a closed form for $g_m$. I found $g_1, g_2$, and a general expression involving a lot of integrals.



    $$
    begin{align}
    g_1=&sum_{ngeq1}frac1{n(n+1)}\
    =&sum_{ngeq1}frac1nint_0^1 t_0^nmathrm{d}t_0\
    =&sum_{ngeq1}int_0^1int_0^{t_0} t_1^{n-1}mathrm{d}t_1mathrm{d}t_0 \
    =&int_0^1int_0^{t_0}sum_{ngeq0}t_1^{n}mathrm{d}t_1mathrm{d}t_0 \
    =&int_0^1int_0^{t_0}frac{mathrm{d}t_1}{1-t_1}mathrm{d}t_0 \
    =&1 \
    end{align}
    $$

    Through the same process, it can be shown that $g_2=frac14$. In fact, through the same process, it can be shown that
    $$g_m=int_0^1int_0^{t_0}int_0^{t_1}cdotsint_0^{t_{m-2}}frac{mathrm{d}t_{m-1}}{1-t_{m-1}}mathrm{d}t_{m-2}cdotsmathrm{d}t_2mathrm{d}t_1mathrm{d}t_0$$
    Which I don't even know how to approach. Any ideas? Thanks.



    This isn't a duplicate question, because I would like to know how to evaluate those nested integrals, if possible.










    share|cite|improve this question











    $endgroup$



    marked as duplicate by Math Lover, Lord Shark the Unknown, user10354138, Masacroso, Brahadeesh Nov 27 '18 at 5:51


    This question has been asked before and already has an answer. If those answers do not fully address your question, please ask a new question.





















      2












      2








      2





      $begingroup$



      This question already has an answer here:




      • Sum of infinite series $sum_{i=1}^{infty} frac 1 {i(i+1)(i+2)…(i+n)}$

        4 answers




      Just for fun, I defined the following series. For $minBbb N$,
      $$g_m=sum_{ngeq1}prod_{i=0}^{m}frac1{n+i}$$
      I am seeking a closed form for $g_m$. I found $g_1, g_2$, and a general expression involving a lot of integrals.



      $$
      begin{align}
      g_1=&sum_{ngeq1}frac1{n(n+1)}\
      =&sum_{ngeq1}frac1nint_0^1 t_0^nmathrm{d}t_0\
      =&sum_{ngeq1}int_0^1int_0^{t_0} t_1^{n-1}mathrm{d}t_1mathrm{d}t_0 \
      =&int_0^1int_0^{t_0}sum_{ngeq0}t_1^{n}mathrm{d}t_1mathrm{d}t_0 \
      =&int_0^1int_0^{t_0}frac{mathrm{d}t_1}{1-t_1}mathrm{d}t_0 \
      =&1 \
      end{align}
      $$

      Through the same process, it can be shown that $g_2=frac14$. In fact, through the same process, it can be shown that
      $$g_m=int_0^1int_0^{t_0}int_0^{t_1}cdotsint_0^{t_{m-2}}frac{mathrm{d}t_{m-1}}{1-t_{m-1}}mathrm{d}t_{m-2}cdotsmathrm{d}t_2mathrm{d}t_1mathrm{d}t_0$$
      Which I don't even know how to approach. Any ideas? Thanks.



      This isn't a duplicate question, because I would like to know how to evaluate those nested integrals, if possible.










      share|cite|improve this question











      $endgroup$





      This question already has an answer here:




      • Sum of infinite series $sum_{i=1}^{infty} frac 1 {i(i+1)(i+2)…(i+n)}$

        4 answers




      Just for fun, I defined the following series. For $minBbb N$,
      $$g_m=sum_{ngeq1}prod_{i=0}^{m}frac1{n+i}$$
      I am seeking a closed form for $g_m$. I found $g_1, g_2$, and a general expression involving a lot of integrals.



      $$
      begin{align}
      g_1=&sum_{ngeq1}frac1{n(n+1)}\
      =&sum_{ngeq1}frac1nint_0^1 t_0^nmathrm{d}t_0\
      =&sum_{ngeq1}int_0^1int_0^{t_0} t_1^{n-1}mathrm{d}t_1mathrm{d}t_0 \
      =&int_0^1int_0^{t_0}sum_{ngeq0}t_1^{n}mathrm{d}t_1mathrm{d}t_0 \
      =&int_0^1int_0^{t_0}frac{mathrm{d}t_1}{1-t_1}mathrm{d}t_0 \
      =&1 \
      end{align}
      $$

      Through the same process, it can be shown that $g_2=frac14$. In fact, through the same process, it can be shown that
      $$g_m=int_0^1int_0^{t_0}int_0^{t_1}cdotsint_0^{t_{m-2}}frac{mathrm{d}t_{m-1}}{1-t_{m-1}}mathrm{d}t_{m-2}cdotsmathrm{d}t_2mathrm{d}t_1mathrm{d}t_0$$
      Which I don't even know how to approach. Any ideas? Thanks.



      This isn't a duplicate question, because I would like to know how to evaluate those nested integrals, if possible.





      This question already has an answer here:




      • Sum of infinite series $sum_{i=1}^{infty} frac 1 {i(i+1)(i+2)…(i+n)}$

        4 answers








      integration sequences-and-series






      share|cite|improve this question















      share|cite|improve this question













      share|cite|improve this question




      share|cite|improve this question








      edited Nov 27 '18 at 5:34







      clathratus

















      asked Nov 27 '18 at 3:27









      clathratusclathratus

      3,767333




      3,767333




      marked as duplicate by Math Lover, Lord Shark the Unknown, user10354138, Masacroso, Brahadeesh Nov 27 '18 at 5:51


      This question has been asked before and already has an answer. If those answers do not fully address your question, please ask a new question.









      marked as duplicate by Math Lover, Lord Shark the Unknown, user10354138, Masacroso, Brahadeesh Nov 27 '18 at 5:51


      This question has been asked before and already has an answer. If those answers do not fully address your question, please ask a new question.
























          2 Answers
          2






          active

          oldest

          votes


















          1












          $begingroup$

          The sum telescopes:
          $$frac{m}{x(x+1)cdots(x+m)}=frac1{x(x+1)cdots(x+m-1)}
          -frac1{(x+1)(x+2)cdots(x+m)}.$$

          Summing this from $1$ to $infty$ gives
          $$msum_{n=1}^inftyfrac{1}{n(n+1)cdots(n+m)}
          =frac1{(m-1)!}$$

          so your sum is $1/m!$.






          share|cite|improve this answer











          $endgroup$





















            1












            $begingroup$

            $sum_{n=1}^{infty}frac{1}{n(n+1)ldots(n+m)}=frac{1}{m}sum_{n=1}^{infty}(frac{1}{n(n+1)ldots(n+m-1)}-frac{1}{(n+1)(n+2)ldots(n+m)})$, which can be easily verified by adding the fractions. From here, you can define $V_{n}=frac{1}{n(n+1)ldots(n+m-1)}$, and so clearly



            $$g_m=frac{1}{m}sum_{n=1}^{infty}(V_n-V_{n+1})$$ and from here it is fairly obvious what to do






            share|cite|improve this answer











            $endgroup$




















              2 Answers
              2






              active

              oldest

              votes








              2 Answers
              2






              active

              oldest

              votes









              active

              oldest

              votes






              active

              oldest

              votes









              1












              $begingroup$

              The sum telescopes:
              $$frac{m}{x(x+1)cdots(x+m)}=frac1{x(x+1)cdots(x+m-1)}
              -frac1{(x+1)(x+2)cdots(x+m)}.$$

              Summing this from $1$ to $infty$ gives
              $$msum_{n=1}^inftyfrac{1}{n(n+1)cdots(n+m)}
              =frac1{(m-1)!}$$

              so your sum is $1/m!$.






              share|cite|improve this answer











              $endgroup$


















                1












                $begingroup$

                The sum telescopes:
                $$frac{m}{x(x+1)cdots(x+m)}=frac1{x(x+1)cdots(x+m-1)}
                -frac1{(x+1)(x+2)cdots(x+m)}.$$

                Summing this from $1$ to $infty$ gives
                $$msum_{n=1}^inftyfrac{1}{n(n+1)cdots(n+m)}
                =frac1{(m-1)!}$$

                so your sum is $1/m!$.






                share|cite|improve this answer











                $endgroup$
















                  1












                  1








                  1





                  $begingroup$

                  The sum telescopes:
                  $$frac{m}{x(x+1)cdots(x+m)}=frac1{x(x+1)cdots(x+m-1)}
                  -frac1{(x+1)(x+2)cdots(x+m)}.$$

                  Summing this from $1$ to $infty$ gives
                  $$msum_{n=1}^inftyfrac{1}{n(n+1)cdots(n+m)}
                  =frac1{(m-1)!}$$

                  so your sum is $1/m!$.






                  share|cite|improve this answer











                  $endgroup$



                  The sum telescopes:
                  $$frac{m}{x(x+1)cdots(x+m)}=frac1{x(x+1)cdots(x+m-1)}
                  -frac1{(x+1)(x+2)cdots(x+m)}.$$

                  Summing this from $1$ to $infty$ gives
                  $$msum_{n=1}^inftyfrac{1}{n(n+1)cdots(n+m)}
                  =frac1{(m-1)!}$$

                  so your sum is $1/m!$.







                  share|cite|improve this answer














                  share|cite|improve this answer



                  share|cite|improve this answer








                  edited Nov 27 '18 at 4:06

























                  answered Nov 27 '18 at 3:39









                  Lord Shark the UnknownLord Shark the Unknown

                  103k1160132




                  103k1160132























                      1












                      $begingroup$

                      $sum_{n=1}^{infty}frac{1}{n(n+1)ldots(n+m)}=frac{1}{m}sum_{n=1}^{infty}(frac{1}{n(n+1)ldots(n+m-1)}-frac{1}{(n+1)(n+2)ldots(n+m)})$, which can be easily verified by adding the fractions. From here, you can define $V_{n}=frac{1}{n(n+1)ldots(n+m-1)}$, and so clearly



                      $$g_m=frac{1}{m}sum_{n=1}^{infty}(V_n-V_{n+1})$$ and from here it is fairly obvious what to do






                      share|cite|improve this answer











                      $endgroup$


















                        1












                        $begingroup$

                        $sum_{n=1}^{infty}frac{1}{n(n+1)ldots(n+m)}=frac{1}{m}sum_{n=1}^{infty}(frac{1}{n(n+1)ldots(n+m-1)}-frac{1}{(n+1)(n+2)ldots(n+m)})$, which can be easily verified by adding the fractions. From here, you can define $V_{n}=frac{1}{n(n+1)ldots(n+m-1)}$, and so clearly



                        $$g_m=frac{1}{m}sum_{n=1}^{infty}(V_n-V_{n+1})$$ and from here it is fairly obvious what to do






                        share|cite|improve this answer











                        $endgroup$
















                          1












                          1








                          1





                          $begingroup$

                          $sum_{n=1}^{infty}frac{1}{n(n+1)ldots(n+m)}=frac{1}{m}sum_{n=1}^{infty}(frac{1}{n(n+1)ldots(n+m-1)}-frac{1}{(n+1)(n+2)ldots(n+m)})$, which can be easily verified by adding the fractions. From here, you can define $V_{n}=frac{1}{n(n+1)ldots(n+m-1)}$, and so clearly



                          $$g_m=frac{1}{m}sum_{n=1}^{infty}(V_n-V_{n+1})$$ and from here it is fairly obvious what to do






                          share|cite|improve this answer











                          $endgroup$



                          $sum_{n=1}^{infty}frac{1}{n(n+1)ldots(n+m)}=frac{1}{m}sum_{n=1}^{infty}(frac{1}{n(n+1)ldots(n+m-1)}-frac{1}{(n+1)(n+2)ldots(n+m)})$, which can be easily verified by adding the fractions. From here, you can define $V_{n}=frac{1}{n(n+1)ldots(n+m-1)}$, and so clearly



                          $$g_m=frac{1}{m}sum_{n=1}^{infty}(V_n-V_{n+1})$$ and from here it is fairly obvious what to do







                          share|cite|improve this answer














                          share|cite|improve this answer



                          share|cite|improve this answer








                          edited Nov 27 '18 at 4:14









                          clathratus

                          3,767333




                          3,767333










                          answered Nov 27 '18 at 3:40









                          SethSeth

                          42812




                          42812















                              Popular posts from this blog

                              Biblatex bibliography style without URLs when DOI exists (in Overleaf with Zotero bibliography)

                              ComboBox Display Member on multiple fields

                              Is it possible to collect Nectar points via Trainline?